Let $AH_1$ and $BH_2$ be the altitudes of triangle $ABC$. Let the tangent to the circumcircle of $ABC$ at $A$ meet $BC$ at point $S_1$, and the tangent at $B$ meet $AC$ at point $S_2$. Let $T_1$ and $T_2$ be the midpoints of $AS_1$ and $BS_2$ respectively. Prove that $T_1T_2$, $AB$ and $H_1H_2$ concur.
Problem
Source: Sharygin CR 2019 P16 (Grade 9 - 11)
Tags: geometry
06.03.2019 15:57
Even I got the same solution
06.03.2019 16:11
Projective cookie. Vrangr wrote: Let $AH_1$ and $BH_2$ be the altitudes of triangle $ABC$. Let the tangent to the circumcircle of $ABC$ at $A$ meet $BC$ at point $S_1$, and the tangent at $B$ meet $AC$ at point $S_2$. Let $T_1$ and $T_2$ be the midpoints of $AS_1$ and $BS_2$ respectively. Prove that $T_1T_2$, $AB$ and $H_1H_2$ concur. Solution . Define $S_3$ and $T_3$ analogously as shown in the diagram. By applying Pascal's theorem on degenerate hexagon $AABBCC,$ it follows that $S_1,S_2,S_3$ are collinear. By Gauss-Bodenmiller Line Theorem on $S_2ABS_1,$ $T_1,T_2,T_3$ are collinear. Let $K_1= BC\cap H_2H_3,$ similarly define $K_2$ and $K_3.$ Applying Desargue's theorem on perspective triangles $H_1H_2H_3$ and $ABC,$ we get $K_1,K_2,K_3$ collinear. Let $P_{\infty}$ be the point at infinity on the line $CS_3.$ It is easy to see that $\overline{K_3H_1H_2}\parallel \overline{CS_3}.$ Now, \[-1=(C,S_3;T_3,P_{\infty})\overset{K_3}{=}(C,B;K_3T_3\cap BC,H_1)\overset{K_3}{=}(C,A;K_3T_3\cap AC,H_2)\]Consider the following lemma: Lemma. Let $ABC$ be a triangle with concurrent cevians $AD,BE,CF.$ Line $EF$ meets $BC$ at $X$ (possibly at a point at infinity). Then $(X,D;B,C)$ is a harmonic bundle. By the above lemma, we know $(C,B;K_1,H_1)=-1$ and $(C,A;K_2,H_2)=-1$. Therefore, $K_3T_3\cap BC=K_1,~K_3T_3\cap AC=K_2,$ so in particular, $K_3,T_3,K_1,K_2$ are collinear, so $T_3\in\overline{K_1K_2K_3},$ similarly $T_1,T_2\in\overline{K_1K_2K_3}.$ Thus, $T_1T_2,AB$ and $H_1H_2$ concur. $\square$
Attachments:

06.03.2019 16:17
Problem Relabeled wrote: Let $BH_2$ and $CH_3$ be the altitudes of triangle $ABC$. Let the tangent to the circumcircle of $ABC$ at $B$ meet $CA$ at point $S_2$, and the tangent at $C$ meet $AB$ at point $S_3$. Let $T_2$ and $T_3$ be the midpoints of $BS_2$ and $CS_3$ respectively. Prove that $T_2T_3$, $AB$ and $H_2H_3$ concur. Let $H_2H_3$ intersect $BC$ at $T$ and $AH_1$ be the third altitude of triangle $ABC$. We proceed with barycentric co-ordinates with sides $BC, CA, AB$ as $a, b, c$ and coordinates of $A (1, 0, 0)$, $B(0, 1, 0)$ and $C(0, 0, 1)$. The coordinate of $H_1$ is, \[H = (0 : a^2 + b^2 - c^2 : a^2 - b^2 + c^2).\]Since $(A, B; H, T)$ is harmonic, coordinate of $T$ is \[T = (0 : - a^2 - b^2 + c^2 : a^2 - b^2 + c^2)\]Since symmedian and $BS_2, CS_3$ are harmonic conjugates by Steiner's Ratio Theorem, \[S_2 = (- a^2 : 0 : c^2)\qquad S_3 = (- a^2 : b^2 : 0).\]\[T_2 = \frac{S_2 + B}{2} = \frac{(-a^2 : 0 : c^2) + (0 : -a^2 + c^2 : 0)}{2} = (-a^2 : -a^2 + c^2 : c^2)\]\[T_3 = (- a^2 : b^2 : -a^2 + b^2).\]Now, to prove $T_2, T_3, T$ to be collinear, it suffices to prove \[\begin{vmatrix}0 & - a^2 - b^2 + c^2 & a^2 - b^2 + c^2 \\ -a^2 & -a^2 + b^2 & b^2 \\ -a^2 & -a^2 + c^2 & c^2 \end{vmatrix} = 0\] However, this follows by subtracting the third row from the first and then adding the second row to the first.
06.03.2019 16:22
This is just USA TSTST 2017 P1
06.03.2019 17:16
Either use USA TSTST 2017 P1 (as above stated) or else: Define $H$ as the orthocenter of $\Delta ABC$, Let $H_3$ be the foot of altitude from $C$ to $AB$, Let $H_2H_3 \cap BC=X_A$, $H_1H_3 \cap AC=X_B$ and $H_1H_2 \cap AB=X_C$, Let $AS_1 \cap X_AX_B=P_1$ and $BS_2 \cap X_AX_B=P_2$ We have $AS_1||H_2H_3$ and similarly, we can have, $BS_2||H_1H_3$, Let $\mathcal{X_1}$ be the infinite point along $AS_1$, $$-1=(C,A;H_2,X_B) \stackrel{X_A}{=} (S_1,A;\mathcal{X_1},P_1)$$hence, $P_1$ is the midpoint of $AS_1$ $\Longrightarrow$ $P_1=T_1$ similarly, we can show that, $P_2=T_2$ and as a result, $T_1,T_2$ lie on the orthic axis, now note that $AB,H_1H_2$ concur on the orthic axis, hence, $T_1T_2,AB$ and $H_1H_2$ are concurrent lines
06.03.2019 18:47
Here’s a non-projective Solution. Trivial angle chasing yields that $T_2H_b$ is tangential to nine-point circle of $\Delta ABC$ and that $T_2H_b = T_2B$ So, $Pow(T_2, (H_aH_bH_c)) = Pow(T_2,(ABC))$. Thus $T_2$ lies on the Orthic axis of $\Delta ABC$. Similarly, $T_1$ lies on the Orthic axis of $\Delta ABC$. So, the required claim follows.
06.03.2019 20:31
Problem wrote: Let $AH_1$ and $BH_2$ be the altitudes of triangle $ABC$. Let the tangent to the circumcircle of $ABC$ at $A$ meet $BC$ at point $S_1$, and the tangent at $B$ meet $AC$ at point $S_2$. Let $T_1$ and $T_2$ be the midpoints of $AS_1$ and $BS_2$ respectively. Prove that $T_1T_2$, $AB$ and $H_1H_2$ concur. Menelaus bash anyone? Let $R=H_1H_2\cap{AB}$, By menelaus theorm on $H_1,H_2,R$ we get $\dfrac{CH_1}{H_1B}\cdot \dfrac{BR}{RA}\cdot \dfrac{AH_2}{H_2C}=1$ .$BH_2=c\cdot{\cos{B}},H_2C=b\cdot{\cos{C}},AH_2=c\cdot{\cos{A}},H_2C=a\cdot{\cos{C}}$$\therefore \dfrac{BR}{RA}=\dfrac{a\cos{B}}{b\cos{A}}=\dfrac{a^2+c^2-b^2}{b^2+c^2-a^2}$ Again by menelaus theorm proving $\dfrac{KS_2}{S_2B}\cdot\dfrac{BR}{RA}\cdot\dfrac{AS_1}{S_1K}=1$ proves $R\in{S_1S_2}$ which solves the problem. Notice that $T_2BA \sim T_2CB$ and $T_1BA \sim T_1AC$,So $T_2B=\dfrac{abc}{a^2-c^2},T_1A=\dfrac{abc}{b^2-c^2}$,Also note that $KA=KB$ and $\angle{KAB}=\angle{KBA}=\angle{C} \implies \dfrac{c}{\sin{180-2C}}=\dfrac{KA}{\sin{C}}$ (from sine rule.) $\implies KA=\dfrac{c}{2\cos{C}}=\dfrac{abc}{a^2+b^2-c^2}$,$KS_2=BS_2-KB= \dfrac{abc}{2(a^2-c^2)}-\dfrac{abc}{a^2+b^2-c^2}=\dfrac{abc(b^2+c^2-a^2)}{2(a^2-c^2)(a^2+b^2-c^2)}$,$S_2B=\dfrac{abc}{2(a^2-c^2)} \implies \dfrac{KS_2}{S_2B}=\dfrac{b^2+c^2-a^2}{a^2+b^2-c^2}$similarly $\dfrac{AS_1}{S_1K}=\dfrac{a^2+b^2-c^2}{a^2+c^2-b^2}$,$\therefore \dfrac{KS_2}{S_2B}\cdot\dfrac{BR}{RA}\cdot\dfrac{AS_1}{S_1K}=\dfrac{b^2+c^2-a^2}{a^2+b^2-c^2}\cdot\dfrac{a^2+c^2-b^2}{b^2+c^2-a^2}\cdot\dfrac{a^2+b^2-c^2}{a^2+c^2-b^2}=1$ as desired.$\blacksquare$
23.03.2019 18:57
Let $P$ $\equiv$ $H_1H_2$ $\cap$ $AB$; $D$, $E$, $F$ be midpoint of $BC$, $CA$, $AB$, resepectively We have: $P_{T_1 / (ABC)}$ = $T_1A^2$ = $\overline{T_1E}$ . $\overline{T_1F}$ = $P_{T_1 / NPC}$, $P_{T_2 / (ABC)}$ = $T_1B^2$ = $\overline{T_1D}$ . $\overline{T_1E}$ = $P_{T_2 / NPC}$, $P_{P / (ABC)}$ = $\overline{PA}$ . $\overline{PB}$ = $\overline{PH_1}$ . $\overline{PH_2}$ = $P_{P / NPC}$ So: $T_1$, $T_2$, $P$ lie on radical axis of ($ABC$) and NPC of $\triangle$ $ABC$ or $AB$, $H_1H_2$, $T_1T_2$ concurrent at $P$
22.11.2019 14:37
Really Nice Problem! Vrangr wrote: Let $AH_1$ and $BH_2$ be the altitudes of triangle $ABC$. Let the tangent to the circumcircle of $ABC$ at $A$ meet $BC$ at point $S_1$, and the tangent at $B$ meet $AC$ at point $S_2$. Let $T_1$ and $T_2$ be the midpoints of $AS_1$ and $BS_2$ respectively. Prove that $T_1T_2$, $AB$ and $H_1H_2$ concur. Solution:- Notations:- Let $P,Q$ be the $A,B$ Expoint of $\triangle ABC$ respectively, $H_3$ is the foot of altitude from $C$ onto $AB$ and $H$ is the orthocenter of $\triangle ABC$. Claim :-$\overline {Q-T_2-T_1-P}$ Let $QP\cap AS_1=R$. So it suffices to prove that $R\equiv T_1$. For this notice that $H_2H_3\|AS_1$. So, $$-1=(A,C;H_2,Q)\overset{P}{=}(A,S_1;\infty_{H_3H_2},R)\implies\boxed{R\equiv T_1}$$Similarly you get $\overline {P-T_2-Q}$. Hence, $\overline {P-T_1-T_2-Q}$. Let $K$ be the $C-\text{expoint}$ of $\triangle ABC$, So, $\overline{P-Q-K}$ as $\triangle ABC$ and $\triangle H_1H_2H_3$ are perspective, hence by Desargues Theorem we are done. So, $T_1T_2\cap AB\equiv K$ which is the $C-\text{expoint}$ of $\triangle ABC$ so $\overline{H_1-H_2-K}$. Hence, $T_1T_2,AB,H_1H_2$ are concurrent at the $C-\text{expoint}$ of $\triangle ABC$. $\blacksquare$.